Optimization - Volume of a Box

Click For Summary
The discussion focuses on finding the dimensions of a rectangular box with maximum volume, constructed from a piece of sheet metal that is twice as long as it is wide and has an area of 800 m². The correct dimensions are derived using algebra, where the width (w) is determined to be 20 m and the length (l) is 40 m. The volume equation is established as V = x(40-2x)(20-2x). Initial calculations made by the user were incorrect due to misinterpretation of the dimensions. The area plays a crucial role in determining the box's dimensions and maximizing its volume.
roman15
Messages
68
Reaction score
0

Homework Statement



Ok I know this question is really easy but for some reason I got it wrong.

You are given a piece of sheet metal that is twice as long as it is wide and has an area of 800m^2. Find the dimensions of the rectangular box that would contain a maximum volume if it were constructed from this piece of metal by cutting squares of equal area at all four corners and folding up the sides. The Box will not have a lid.


Homework Equations





The Attempt at a Solution


So basically I drew the piece of metal, one side was 2m the other was 1m, then I subtracted the corners, so I had 2-2x=1-x and 1-2x and the height being x
so V=x(1-x)(1-2x)
=2x^3-3x^2+x
then V'=6x^2-6x+1
but that didnt give me the right answer...Im not sure, but does the area have any significance to the problem?
 
Physics news on Phys.org
Yes. All of your logic for the calculus problem was right, except for the fact that one side does not equal 2m and the other does not equal 1 m. You have to use algebra to figure you the length and width of the rectangular box.

width = w
length = l
l = 2w
800 = 2w^2
400 = w^2
w = 20
l = 40

So the equation above should be
V = x(40-2x)(20-2x)
 
Question: A clock's minute hand has length 4 and its hour hand has length 3. What is the distance between the tips at the moment when it is increasing most rapidly?(Putnam Exam Question) Answer: Making assumption that both the hands moves at constant angular velocities, the answer is ## \sqrt{7} .## But don't you think this assumption is somewhat doubtful and wrong?

Similar threads

Replies
1
Views
2K
  • · Replies 9 ·
Replies
9
Views
2K
  • · Replies 4 ·
Replies
4
Views
2K
  • · Replies 25 ·
Replies
25
Views
2K
  • · Replies 18 ·
Replies
18
Views
2K
  • · Replies 1 ·
Replies
1
Views
2K
  • · Replies 3 ·
Replies
3
Views
4K
  • · Replies 17 ·
Replies
17
Views
4K
  • · Replies 11 ·
Replies
11
Views
6K
  • · Replies 2 ·
Replies
2
Views
2K